Which one of the following is a complete and accurate list of clerks, any one of whom could stock aisle 3?

kens on September 9, 2020

June 2001 LSAT lg#3

Why can't kurt be in aisle 3? LMKKMOJLJ? Thanks in advance!

Reply
Create a free account to read and take part in forum discussions.

Already have an account? log in

Victoria on September 25, 2020

Hi @kenken,

Happy to help!

We know that K must stock Aisle 2.

Rule 5 tells us that K stocks the only aisle between the two aisles that M stocks.

So, there are two issues with your above setup:

1) K does not stock Aisle 2.
2) There are two aisles in between the two aisles that M stocks.

We cannot have K stock Aisle 3 because it would simultaneously break Rules 3 and 5. If K stocked Aisle 3, M would have to stock Aisles 1 and 4. However, we know from Rule 3 that M cannot stock Aisle 1 and we know from Rule 5 that there is only one aisle between the two aisles that M stocks, not two.

Hope this helps! Please let us know if you have any other questions.